Difference between revisions of "1990 AHSME Problems/Problem 29"

(Solution)
Line 11: Line 11:
 
== Solution ==
 
== Solution ==
 
<math>\fbox{D}</math>
 
<math>\fbox{D}</math>
 +
Notice that inclusion of the integers between 34 to 100 inclusive is allowed as long as no integer between 11 and 33 inclusive is within the set. This provides a total of 100 - 34 + 1 = 67 solutions.
 +
 +
Further analyzing the remaining integers between 1 and 10, we notice that we can include all the numbers except 3 (as including 3 would force us to remove both 9 and 1) to obtain the maximum number of  9 solutions.
 +
 +
Thus, 67 + 9 = 76 (D)
  
 
== See also ==
 
== See also ==

Revision as of 20:24, 24 August 2015

Problem

A subset of the integers $1,2,\cdots,100$ has the property that none of its members is 3 times another. What is the largest number of members such a subset can have?

$\text{(A) } 50\quad \text{(B) } 66\quad \text{(C) } 67\quad \text{(D) } 76\quad \text{(E) } 78$

Solution

$\fbox{D}$ Notice that inclusion of the integers between 34 to 100 inclusive is allowed as long as no integer between 11 and 33 inclusive is within the set. This provides a total of 100 - 34 + 1 = 67 solutions.

Further analyzing the remaining integers between 1 and 10, we notice that we can include all the numbers except 3 (as including 3 would force us to remove both 9 and 1) to obtain the maximum number of 9 solutions.

Thus, 67 + 9 = 76 (D)

See also

1990 AHSME (ProblemsAnswer KeyResources)
Preceded by
Problem 28
Followed by
Problem 29
1 2 3 4 5 6 7 8 9 10 11 12 13 14 15 16 17 18 19 20 21 22 23 24 25 26 27 28 29 30
All AHSME Problems and Solutions

The problems on this page are copyrighted by the Mathematical Association of America's American Mathematics Competitions. AMC logo.png